Expressão de prova para a função de autocovariância de AR (1)

A representação para o modelo AR (1) é a seguinte:

$ Y_t = c + ϕY_ {t-1} + ε_t $

onde $ c = (1 -ϕ) μ $ ( $ c $ é uma constante).


Quero entender os cálculos que existem estão por trás da fórmula geral da autocovariância de AR (1), que é $ γ (h) = \ operatorname {Var} (Y_t) ⋅ϕ ^ {| h |} $

Até agora, executei as seguintes etapas – comecei com $ γ (1) $ :

$ \ operatorname {Cov} (Y_t, Y_ {t-1}) = γ (1) = $

$ = \ operatorname {Cov} (ϕY_ {t-1} + ε_t, ϕY_ {t-2} + ε_t) = $

$ = \ operatorname {Cov} (ϕY_ {t-1}, ϕY_ {t-2}) + \ operatorname {Cov} (ϕY_ {t-1}, ε_t) + \ operatorname {Cov} ( ε_t, ϕY_ {t-2}) + \ operatorname {Cov} (ε_t , ε_t) $

$ γ (1) = ϕ ^ 2γ (1) + ??? + ??? + σ ^ 2 $

Como você pode ver, a partir deste ponto não posso continuar porque não sei quais são os valores de $ \ operatorname {Cov} (ϕY_ {t-1}, ε_t) $ e $ \ operatorname {Cov} ( ε_t, ϕY_ {t-2}) $


Qualquer ajuda será muito apreciada. Desde já, obrigado.

Resposta

Vamos escrever $ \ gamma ( 1) $ : $$ \ begin {align} \ gamma (1) & = cov (Y_t, Y_ {t -1}) = cov (c + \ phi Y_ {t-1} + \ epsilon_t, Y_ {t-1}) \\ & = cov (c, Y_ {t- 1}) + \ phi cov (Y_ {t-1}, Y_ {t-1}) + cov (\ epsilon_t, Y_ {t-1}) \\ & = \ phi \ gamma (0) \ end {align} $$

uma vez que $ cov (c, Y_ {t-1}) = cov (\ epsilon_t, Y_ {t-1}) = 0 $ , (ou seja, a saída anterior é independente da entrada futura).

Da mesma forma, $ \ gamma (2) = cov (Y_t, Y_ {t-2}) = cov (\ phi Y_ {t-1} + c + \ epsilon_t, Y_ {t-2}) = \ phi \ gamma (1) = \ phi ^ 2 \ gamma (0) $ .

Se continuarmos dessa forma, obteremos $ cov (Y_t, Y_ {th}) = \ phi \ gamma (h-1) = … \ phi ^ h \ gamma (0) $ , onde $ h \ geq0 $ . Generalizando para rendimentos $ h $ negativos $ \ gamma (h) = \ phi ^ {| h |} \ gamma (0) $ , onde $ \ gamma (0 ) = var (Y_t) $ .

PS toda essa análise assume que $ \ epsilon_t $ é WSS, portanto $ y_t $ da propriedade de filtragem de LTI.

Comentários

  • há um erro de digitação na primeira linha .. sinal de identidade colocado incorretamente.
  • Na primeira linha, eu faria substitua o terceiro ” + ” sinal pelo ” = ” sinal: $ cov (c + \ phi Y_ {t-1} + \ epsilon_t, Y_ {t-1}) = cov (c, Y_ {t-1}) + \ phi cov (Y_ { t-1}, Y_ {t-1}) + cov (\ epsilon_t, Y_ {t-1}) $
  • Ao tentar editar o erro de digitação endereçado por @Jesper, converti aquele sinal específico = para + assinar, e tornou mais errado :). Vejo que a razão é por causa da renderização. Embora a ordem das declarações tex estejam corretas, elas foram exibidas em uma ordem diferente. De qualquer forma, eu ‘ usei as declarações de alinhamento e deixei isso muito mais claro. Espero que ‘ esteja tudo bem.
  • A expressão para autocovariância condicional é a mesma? Ou seja, $ Cov [Y_ {t_0 + n + h}, Y_ {t_0 + n} | F_ {t_0}] = \ phi ^ h Var [Y_ {t_0 + n} | F_ {t_0}] = \ phi ^ h \ sigma ^ 2 \ frac {1- \ phi ^ {2n}} {1- \ phi ^ 2} $ espera?

Resposta

Começando com o que você forneceu:

$ y_ {t} = c + \ phi y_ {t-1} + \ epsilon_ {t} \ tag {1} $

Onde $ c = (1 – \ phi) \ mu $


Podemos reescrever $ (1) $ as:

\ begin {array} \ y_ {t} & = & c + \ phi y_ {t-1} + \ epsilon_ {t} \\ & = & (1 – \ phi) \ mu + \ phi y_ {t-1} + \ epsilon_ {t} \\ & = & \ mu – \ phi \ mu + \ phi y_ {t-1} + \ epsilon_ {t} \\ \ end {array}

Então,

$ y_ {t} – \ mu = \ phi (y_ {t-1} – \ mu) + \ epsilon_ {t} \ tag {2} $

Se deixarmos $ \ tilde {y_ {t}} = y_ {t} – \ mu $ , então a equação $ (2) $ pode ser escrito como:

$ \ tilde {y} _ {t} = \ phi \ tilde {y} _ {t-1} + \ epsilon_ {t} \ tag {3} $


Variância

A variação de $ (3) $ é obtido elevando a expressão ao quadrado e tomando as expectativas, que termina em:

\ begin { matriz} \ \ tilde {y} _ {t} ^ 2 & = & (\ phi \ tilde {y} _ {t -1} + \ epsilon_ {t}) ^ 2 \\ & = & (\ phi \ tilde {y} _ {t -1}) ^ 2 + 2 \ phi \ tilde {y} _ {t-1} \ epsilon_ {t} + (\ epsilon_ {t}) ^ 2 \\ & = & \ phi ^ {2} \ tilde {y} _ {t-1} ^ {2} + 2 \ phi \ tilde {y} _ {t-1} \ epsilon_ {t} + \ epsilon_ {t} ^ 2 \ end {array}

Agora considere a expectativa:

$ E (\ tilde {y} _ {t} ^ 2) = \ phi ^ {2} E (\ tilde {y} _ {t-1} ^ {2}) + 2 \ phi E (\ tilde {y } _ {t-1} \ epsilon_ {t}) + E (\ epsilon_ {t} ^ 2) $

Ela e chamaremos:

  • $ \ sigma_ {y} ^ {2} $ é a variância do processo estacionário.
  • O segundo termo no lado direito da equação é zero porque $ \ tilde {y} _ {t-1} $ e $ \ epsilon_ {t} $ são independentes e ambos têm expectativa nula.
  • O último termo à direita é a variância da inovação, denotada como $ \ sigma ^ {2} $ (observe que não há subscrito para isso).

Finalmente,

$ \ sigma_ {y} ^ {2} = \ phi ^ {2} \ sigma_ { y} ^ {2} + \ sigma ^ {2} $

Se resolvermos a variância do processo, a saber $ \ sigma_ { y} ^ {2} $ , temos:

$ \ sigma_ {y} ^ {2} = \ frac {\ sigma ^ 2 } {1 – \ phi ^ 2} \ tag {4} $


Autocovariância

Vamos usar o mesmo truque que usamos para a fórmula $ (3) $ . A autocovariância entre as observações separadas por $ h $ períodos é então:

\ begin {array} \ \ gamma_ {h} & = & E [(y_ {t – h} – \ mu) (y_ {t} – \ mu)] \\ & = & E [(\ tilde {y} _ {t – h}) (\ tilde {y } _ {t})] \\ & = & E [\ tilde {y} _ {t – h} (\ phi \ tilde {y} _ {t – 1} + \ epsilon_ {t}) \\ \ end {array}

As inovações não estão correlacionadas com os valores anteriores da série, então $ E [\ tilde {y} _ {th} \ epsilon_ {t}] = 0 $ e ficamos com:

$ \ gamma_ {h} = \ phi \ gamma_ {h-1} \ tag {5} $

Para $ h = 1, 2, \ ldots $ e com $ \ gamma_ {0} = \ sigma_ {y} ^ 2 $


Para o caso particular de $ AR (1) $ , a equação $ (5) $ torna-se:

$ \ gamma_ {1} = \ phi \ gamma_ {0} $

E usando o resultado da equação $ (4) $ : $ \ gamma_ {0} = \ sigma_ {y} ^ {2} = \ frac {\ sigma ^ 2} {1 – \ phi ^ 2} $ que acabamos com

$ \ gamma_ {1} = \ frac {\ sigma ^ 2} {1 – \ phi ^ 2} \ phi $


Fonte original: slides de Andrés M. Alonso & Carolina García-Martos slides. Disponível aqui: http://www.est.uc3m.es/amalonso/esp/TSAtema4petten.pdf

Deixe uma resposta

O seu endereço de email não será publicado. Campos obrigatórios marcados com *